Question and Answers Forum

All Questions      Topic List

None Questions

Previous in All Question      Next in All Question      

Previous in None      Next in None      

Question Number 137203 by mathocean1 last updated on 31/Mar/21

∫ ((ln(1+x))/x)=?

$$\int\:\frac{{ln}\left(\mathrm{1}+{x}\right)}{{x}}=? \\ $$

Answered by Dwaipayan Shikari last updated on 31/Mar/21

∫((log(1+x))/x)dx  =Σ_(n=1) ^∞ (−1)^(n+1) ∫(x^(n−1) /n)=Σ_(n=1) ^∞ (((−1)^(n+1) x^n )/n^2 )

$$\int\frac{{log}\left(\mathrm{1}+{x}\right)}{{x}}{dx} \\ $$$$=\underset{{n}=\mathrm{1}} {\overset{\infty} {\sum}}\left(−\mathrm{1}\right)^{{n}+\mathrm{1}} \int\frac{{x}^{{n}−\mathrm{1}} }{{n}}=\underset{{n}=\mathrm{1}} {\overset{\infty} {\sum}}\frac{\left(−\mathrm{1}\right)^{{n}+\mathrm{1}} {x}^{{n}} }{{n}^{\mathrm{2}} } \\ $$

Answered by Ar Brandon last updated on 31/Mar/21

I=∫_0 ^1 ((ln(1+x))/x)dx=∫_0 ^1 (1/x)Σ_(n=0) ^∞ (−1)^n (x^(n+1) /((n+1)))dx     =Σ_(n=0) ^∞ (((−1)^n )/(n+1))∫_0 ^1 x^n dx=Σ_(n=0) ^∞ (((−1)^n )/((n+1)^2 ))=Σ_(n=1) ^∞ (((−1)^(n+1) )/n^2 )     =2Σ_(n=0) ^∞ (1/((2n+1)^2 ))−Σ_(n=1) ^∞ (1/n^2 )=2×(3/4)ζ(2)−ζ(2)=(1/2)ζ(2)=(π^2 /(12))

$$\mathcal{I}=\int_{\mathrm{0}} ^{\mathrm{1}} \frac{\mathrm{ln}\left(\mathrm{1}+\mathrm{x}\right)}{\mathrm{x}}\mathrm{dx}=\int_{\mathrm{0}} ^{\mathrm{1}} \frac{\mathrm{1}}{\mathrm{x}}\underset{\mathrm{n}=\mathrm{0}} {\overset{\infty} {\sum}}\left(−\mathrm{1}\right)^{\mathrm{n}} \frac{\mathrm{x}^{\mathrm{n}+\mathrm{1}} }{\left(\mathrm{n}+\mathrm{1}\right)}\mathrm{dx} \\ $$$$\:\:\:=\underset{\mathrm{n}=\mathrm{0}} {\overset{\infty} {\sum}}\frac{\left(−\mathrm{1}\right)^{\mathrm{n}} }{\mathrm{n}+\mathrm{1}}\int_{\mathrm{0}} ^{\mathrm{1}} \mathrm{x}^{\mathrm{n}} \mathrm{dx}=\underset{\mathrm{n}=\mathrm{0}} {\overset{\infty} {\sum}}\frac{\left(−\mathrm{1}\right)^{\mathrm{n}} }{\left(\mathrm{n}+\mathrm{1}\right)^{\mathrm{2}} }=\underset{\mathrm{n}=\mathrm{1}} {\overset{\infty} {\sum}}\frac{\left(−\mathrm{1}\right)^{\mathrm{n}+\mathrm{1}} }{\mathrm{n}^{\mathrm{2}} } \\ $$$$\:\:\:=\mathrm{2}\underset{\mathrm{n}=\mathrm{0}} {\overset{\infty} {\sum}}\frac{\mathrm{1}}{\left(\mathrm{2n}+\mathrm{1}\right)^{\mathrm{2}} }−\underset{\mathrm{n}=\mathrm{1}} {\overset{\infty} {\sum}}\frac{\mathrm{1}}{\mathrm{n}^{\mathrm{2}} }=\mathrm{2}×\frac{\mathrm{3}}{\mathrm{4}}\zeta\left(\mathrm{2}\right)−\zeta\left(\mathrm{2}\right)=\frac{\mathrm{1}}{\mathrm{2}}\zeta\left(\mathrm{2}\right)=\frac{\pi^{\mathrm{2}} }{\mathrm{12}} \\ $$

Terms of Service

Privacy Policy

Contact: info@tinkutara.com